Is Qij=AiBj a Tensor of Rank 2?

Click For Summary
The discussion centers on proving that the object Q, defined as Qij=AiBj, is a tensor of rank 2. Participants explore how to demonstrate that Q transforms correctly under rotations, using the transformation properties of vectors A and B. The key insight is that if Q transforms according to the tensor transformation law, it qualifies as a tensor. The transformation equations are derived, confirming that Q retains its form under rotation. Ultimately, the conclusion is that an object is classified as a tensor if it transforms like one.
flintbox
Messages
10
Reaction score
0

Homework Statement


Suppose A and B are vectors. Show that the object Q with nine components Qij=AiBj is a tensor of rank 2.

Homework Equations


A tensor transforms under rotations (R) as a vector:
Tij'=RinRjmTnm

The Attempt at a Solution


I wanted to just create the matrix, but I don't know how to prove that this is also a tensor.
 
Physics news on Phys.org
flintbox said:

Homework Statement


Suppose A and B are vectors. Show that the object Q with nine components Qij=AiBj is a tensor of rank 2.

Homework Equations


A tensor transforms under rotations (R) as a vector:
Tij'=RinRjmTnm

The Attempt at a Solution


I wanted to just create the matrix, but I don't know how to prove that this is also a tensor.
Well, how do the vectors ##A## and ##B## transform under rotations?
 
  • Like
Likes BvU
Thanks a lot!
I think I understand it now:
$$A_i' B_j' = R_{in}A_n R_{jm}A_m$$
$$A_i' B_j' = R_{in} R_{jm} (A_n A_m)$$
$$A_i' B_j' = R_{in} R_{jm} Q_{nm}$$
$$A_i' B_j' = Q'_{nm}$$
So we for proving something is a tensor, we just apply some transformations to it, right?
 
flintbox said:
Thanks a lot!
I think I understand it now:
$$A_i' B_j' = R_{in}A_n R_{jm}A_m$$
$$A_i' B_j' = R_{in} R_{jm} (A_n A_m)$$
$$A_i' B_j' = R_{in} R_{jm} Q_{nm}$$
$$A_i' B_j' = Q'_{nm}$$
So we for proving something is a tensor, we just apply some transformations to it, right?

Yes, something is a tensor if it transforms like a tensor.
 
Question: A clock's minute hand has length 4 and its hour hand has length 3. What is the distance between the tips at the moment when it is increasing most rapidly?(Putnam Exam Question) Answer: Making assumption that both the hands moves at constant angular velocities, the answer is ## \sqrt{7} .## But don't you think this assumption is somewhat doubtful and wrong?

Similar threads

  • · Replies 1 ·
Replies
1
Views
4K
  • · Replies 2 ·
Replies
2
Views
3K
  • · Replies 1 ·
Replies
1
Views
1K
  • · Replies 4 ·
Replies
4
Views
2K
  • · Replies 5 ·
Replies
5
Views
2K
Replies
1
Views
2K
  • · Replies 8 ·
Replies
8
Views
2K
  • · Replies 1 ·
Replies
1
Views
2K
  • · Replies 1 ·
Replies
1
Views
1K
  • · Replies 3 ·
Replies
3
Views
2K